Continuous function and limit $ f(x,y)=left{begin{matrix} (x-y)sinfrac{1}{x}sinfrac{1}{y} & , xyneq 0 \ 0...

List of Python versions

Seeking colloquialism for “just because”

How to bypass password on Windows XP account?

What is the role of the transistor and diode in a soft start circuit?

How discoverable are IPv6 addresses and AAAA names by potential attackers?

Sci-Fi book where patients in a coma ward all live in a subconscious world linked together

Can a non-EU citizen traveling with me come with me through the EU passport line?

Why is "Consequences inflicted." not a sentence?

In predicate logic, does existential quantification (∃) include universal quantification (∀), i.e. can 'some' imply 'all'?

Identify plant with long narrow paired leaves and reddish stems

How does debian/ubuntu knows a package has a updated version

Why do we bend a book to keep it straight?

Coloring maths inside a tcolorbox

How to tell that you are a giant?

Identifying polygons that intersect with another layer using QGIS?

Error "illegal generic type for instanceof" when using local classes

What does an IRS interview request entail when called in to verify expenses for a sole proprietor small business?

Why light coming from distant stars is not discreet?

3 doors, three guards, one stone

How widely used is the term Treppenwitz? Is it something that most Germans know?

How do I stop a creek from eroding my steep embankment?

Is it ethical to give a final exam after the professor has quit before teaching the remaining chapters of the course?

Fundamental Solution of the Pell Equation

Why did the IBM 650 use bi-quinary?



Continuous function and limit $ f(x,y)=left{begin{matrix} (x-y)sinfrac{1}{x}sinfrac{1}{y} & , xyneq 0 \ 0 &, x=y=0 end{matrix}right. $



Announcing the arrival of Valued Associate #679: Cesar Manara
Planned maintenance scheduled April 17/18, 2019 at 00:00UTC (8:00pm US/Eastern)Strange behavior of $lim_{xto0}frac{sinleft(xsinleft(frac1xright)right)}{xsinleft(frac1xright)}$continuity and limits of $f(x,y)= begin{cases} frac{yln(x+1)}{y^2+(ln(x+1))^2} &text{if $y neq 0$ }\0&text{if $ y=0$}end{cases}$Check whether the given function is differentiable at $(0,0)$Left hand and right hand derivatives of $xsin ({1over x})$ at $0$Limit of the function $lim limits_{(x,y)to (0,0)} sin (frac{x^2}{x+y}) (x+y neq 0)$.Limit and continuity of multivariable functionShow that f is continous at $(0,0)$ where $ f(x,y)= ysin(frac{1}{x})$ if $x neq 0$ else $0$Find if two variables function is differentiable: $f(x,y)=frac{x(1-cos x)}{sin(x^2+y^2)}$find: $lim_{nrightarrowinfty}frac{sinleft(x+frac{1}{n}right)-sinleft(xright)}{sinleft(x+frac{1}{n}right)}$Continuity of $begin{cases}(xy+y^2)/(x^4+y^2)&text{if }(x,y)neq(0,0),\0&text{if }(x,y)=(0,0)end{cases}$ at origin using polar coordinates












2












$begingroup$


I have this function:$$ f(x,y)=left{begin{matrix}
(x-y)sinfrac{1}{x}sinfrac{1}{y} & , xyneq 0 \ 0
&, x=y=0
end{matrix}right. $$

a) Show that $ lim_{xrightarrow 0}[lim_{yrightarrow 0} f(x,y)] $ does not exist.



b) Show that $ f(x,y) $ is continuous at $ (0,0) $ .



For (a) I took $ lim_{yrightarrow 0} f(x,y) $ and I got that it's equal to $$ xsinfrac{1}{x}lim_{yrightarrow 0}sinfrac{1}{y} - sinfrac{1}{x}lim_{yrightarrow 0}y sinfrac{1}{y} $$, but $$ lim_{yrightarrow 0}sinfrac{1}{y} $$ does not exist, so $ lim_{yrightarrow 0} f(x,y) $ does not exist and $ lim_{xrightarrow 0}[lim_{yrightarrow 0} f(x,y)] $ does not exist. Am I right?



For (b) I guess that I have to show that $$ lim_{(x,y)rightarrow (0,0)} f(x,y) = 0 $$ How can I do this ?










share|cite|improve this question











$endgroup$

















    2












    $begingroup$


    I have this function:$$ f(x,y)=left{begin{matrix}
    (x-y)sinfrac{1}{x}sinfrac{1}{y} & , xyneq 0 \ 0
    &, x=y=0
    end{matrix}right. $$

    a) Show that $ lim_{xrightarrow 0}[lim_{yrightarrow 0} f(x,y)] $ does not exist.



    b) Show that $ f(x,y) $ is continuous at $ (0,0) $ .



    For (a) I took $ lim_{yrightarrow 0} f(x,y) $ and I got that it's equal to $$ xsinfrac{1}{x}lim_{yrightarrow 0}sinfrac{1}{y} - sinfrac{1}{x}lim_{yrightarrow 0}y sinfrac{1}{y} $$, but $$ lim_{yrightarrow 0}sinfrac{1}{y} $$ does not exist, so $ lim_{yrightarrow 0} f(x,y) $ does not exist and $ lim_{xrightarrow 0}[lim_{yrightarrow 0} f(x,y)] $ does not exist. Am I right?



    For (b) I guess that I have to show that $$ lim_{(x,y)rightarrow (0,0)} f(x,y) = 0 $$ How can I do this ?










    share|cite|improve this question











    $endgroup$















      2












      2








      2





      $begingroup$


      I have this function:$$ f(x,y)=left{begin{matrix}
      (x-y)sinfrac{1}{x}sinfrac{1}{y} & , xyneq 0 \ 0
      &, x=y=0
      end{matrix}right. $$

      a) Show that $ lim_{xrightarrow 0}[lim_{yrightarrow 0} f(x,y)] $ does not exist.



      b) Show that $ f(x,y) $ is continuous at $ (0,0) $ .



      For (a) I took $ lim_{yrightarrow 0} f(x,y) $ and I got that it's equal to $$ xsinfrac{1}{x}lim_{yrightarrow 0}sinfrac{1}{y} - sinfrac{1}{x}lim_{yrightarrow 0}y sinfrac{1}{y} $$, but $$ lim_{yrightarrow 0}sinfrac{1}{y} $$ does not exist, so $ lim_{yrightarrow 0} f(x,y) $ does not exist and $ lim_{xrightarrow 0}[lim_{yrightarrow 0} f(x,y)] $ does not exist. Am I right?



      For (b) I guess that I have to show that $$ lim_{(x,y)rightarrow (0,0)} f(x,y) = 0 $$ How can I do this ?










      share|cite|improve this question











      $endgroup$




      I have this function:$$ f(x,y)=left{begin{matrix}
      (x-y)sinfrac{1}{x}sinfrac{1}{y} & , xyneq 0 \ 0
      &, x=y=0
      end{matrix}right. $$

      a) Show that $ lim_{xrightarrow 0}[lim_{yrightarrow 0} f(x,y)] $ does not exist.



      b) Show that $ f(x,y) $ is continuous at $ (0,0) $ .



      For (a) I took $ lim_{yrightarrow 0} f(x,y) $ and I got that it's equal to $$ xsinfrac{1}{x}lim_{yrightarrow 0}sinfrac{1}{y} - sinfrac{1}{x}lim_{yrightarrow 0}y sinfrac{1}{y} $$, but $$ lim_{yrightarrow 0}sinfrac{1}{y} $$ does not exist, so $ lim_{yrightarrow 0} f(x,y) $ does not exist and $ lim_{xrightarrow 0}[lim_{yrightarrow 0} f(x,y)] $ does not exist. Am I right?



      For (b) I guess that I have to show that $$ lim_{(x,y)rightarrow (0,0)} f(x,y) = 0 $$ How can I do this ?







      limits functions trigonometry continued-fractions






      share|cite|improve this question















      share|cite|improve this question













      share|cite|improve this question




      share|cite|improve this question








      edited Mar 6 at 0:54









      rash

      568216




      568216










      asked Mar 6 at 0:03









      Dr.MathematicsDr.Mathematics

      516




      516






















          3 Answers
          3






          active

          oldest

          votes


















          2












          $begingroup$

          For part (b), I think the easiest proof is just to note that $$|(x-y)sinfrac{1}{x}sinfrac{1}{y}| leq |x-y|.$$



          Thus, it's enough to prove that $$lim_{(x,y) to (0,0)}(x-y) = 0.$$






          share|cite|improve this answer









          $endgroup$





















            1












            $begingroup$

            Use polar coordinates $x=rcostheta$ and $y=rsintheta$. The limit is the same as $rto 0$. The original trigonometric functions are always between $-1$ and $1$, $|costheta-sintheta|<2$, so your original expression is always less than $2r$ in absolute value.






            share|cite|improve this answer









            $endgroup$













            • $begingroup$
              For (a), you're not quite where you need to be. You need both that the limit of one term of your difference does not exist and also that the limit of the other term does exist. For example, $lim_{x to 0} (sin (1/x) - sin (1/x)) = 0$ even though neither term of the difference has a limit.
              $endgroup$
              – Robert Shore
              Mar 6 at 0:28










            • $begingroup$
              Sorry, I should have specified that this is the answer for part b only. Part a is ok.
              $endgroup$
              – Andrei
              Mar 6 at 0:30










            • $begingroup$
              @RobertShore yeah I am sorry. In my paper I showed that $ ysinfrac{1}{y}rightarrow 0 $
              $endgroup$
              – Dr.Mathematics
              Mar 6 at 1:00



















            0












            $begingroup$

            $sin{(1/x)sin(1/y)}$ is an interval and $x-y$ under the limit of $(x,y)to{(0,0)}$ is an interval too. if a function in a close interval is continuous, its valued field is an interval, but its converse proposition may be not true. for example, your function is a counterexample. so you should separate your problem into two cases with $f^{'}(0,0)$ and $lim_{(x,y)to{(0,0)}}f^{'}(x,y)$



            the first limit exists but the second do not! for example rewrite your function to $F_{x}^{'}(x,y)=sin(1/x)sin(1/y)+(x-y)sin(1/y)cos(1/x)(-1/x^2)$



            $F_{y}^{'}(x,y)=-sin(1/x)sin(1/y)+(x-y)sin(1/x)cos(1/y)(-1/y^2)$



            divide both side with $sin(1/x)sin(1/y)$ to get



            $1+(x-y)cos(1/x)(-frac{1}{x^{2}sin(1/x)})$



            $-1+(x-y)cos(1/y)(-frac{1}{y^{2}sin(1/y))}$



            since $f(x)=x^2{sin(1/x)}$ or equal to zero exists a limit at $f^{'}(0)$, but $lim_{xto{0}}f^{'}(x)=2xsin(1/x)-cos(1/x)$ do not exists, this result give a certain conclusion to your problem. thank you!






            share|cite|improve this answer









            $endgroup$














              Your Answer








              StackExchange.ready(function() {
              var channelOptions = {
              tags: "".split(" "),
              id: "69"
              };
              initTagRenderer("".split(" "), "".split(" "), channelOptions);

              StackExchange.using("externalEditor", function() {
              // Have to fire editor after snippets, if snippets enabled
              if (StackExchange.settings.snippets.snippetsEnabled) {
              StackExchange.using("snippets", function() {
              createEditor();
              });
              }
              else {
              createEditor();
              }
              });

              function createEditor() {
              StackExchange.prepareEditor({
              heartbeatType: 'answer',
              autoActivateHeartbeat: false,
              convertImagesToLinks: true,
              noModals: true,
              showLowRepImageUploadWarning: true,
              reputationToPostImages: 10,
              bindNavPrevention: true,
              postfix: "",
              imageUploader: {
              brandingHtml: "Powered by u003ca class="icon-imgur-white" href="https://imgur.com/"u003eu003c/au003e",
              contentPolicyHtml: "User contributions licensed under u003ca href="https://creativecommons.org/licenses/by-sa/3.0/"u003ecc by-sa 3.0 with attribution requiredu003c/au003e u003ca href="https://stackoverflow.com/legal/content-policy"u003e(content policy)u003c/au003e",
              allowUrls: true
              },
              noCode: true, onDemand: true,
              discardSelector: ".discard-answer"
              ,immediatelyShowMarkdownHelp:true
              });


              }
              });














              draft saved

              draft discarded


















              StackExchange.ready(
              function () {
              StackExchange.openid.initPostLogin('.new-post-login', 'https%3a%2f%2fmath.stackexchange.com%2fquestions%2f3136903%2fcontinuous-function-and-limit-fx-y-left-beginmatrix-x-y-sin-frac1x%23new-answer', 'question_page');
              }
              );

              Post as a guest















              Required, but never shown

























              3 Answers
              3






              active

              oldest

              votes








              3 Answers
              3






              active

              oldest

              votes









              active

              oldest

              votes






              active

              oldest

              votes









              2












              $begingroup$

              For part (b), I think the easiest proof is just to note that $$|(x-y)sinfrac{1}{x}sinfrac{1}{y}| leq |x-y|.$$



              Thus, it's enough to prove that $$lim_{(x,y) to (0,0)}(x-y) = 0.$$






              share|cite|improve this answer









              $endgroup$


















                2












                $begingroup$

                For part (b), I think the easiest proof is just to note that $$|(x-y)sinfrac{1}{x}sinfrac{1}{y}| leq |x-y|.$$



                Thus, it's enough to prove that $$lim_{(x,y) to (0,0)}(x-y) = 0.$$






                share|cite|improve this answer









                $endgroup$
















                  2












                  2








                  2





                  $begingroup$

                  For part (b), I think the easiest proof is just to note that $$|(x-y)sinfrac{1}{x}sinfrac{1}{y}| leq |x-y|.$$



                  Thus, it's enough to prove that $$lim_{(x,y) to (0,0)}(x-y) = 0.$$






                  share|cite|improve this answer









                  $endgroup$



                  For part (b), I think the easiest proof is just to note that $$|(x-y)sinfrac{1}{x}sinfrac{1}{y}| leq |x-y|.$$



                  Thus, it's enough to prove that $$lim_{(x,y) to (0,0)}(x-y) = 0.$$







                  share|cite|improve this answer












                  share|cite|improve this answer



                  share|cite|improve this answer










                  answered Mar 6 at 0:36









                  Robert ShoreRobert Shore

                  3,832324




                  3,832324























                      1












                      $begingroup$

                      Use polar coordinates $x=rcostheta$ and $y=rsintheta$. The limit is the same as $rto 0$. The original trigonometric functions are always between $-1$ and $1$, $|costheta-sintheta|<2$, so your original expression is always less than $2r$ in absolute value.






                      share|cite|improve this answer









                      $endgroup$













                      • $begingroup$
                        For (a), you're not quite where you need to be. You need both that the limit of one term of your difference does not exist and also that the limit of the other term does exist. For example, $lim_{x to 0} (sin (1/x) - sin (1/x)) = 0$ even though neither term of the difference has a limit.
                        $endgroup$
                        – Robert Shore
                        Mar 6 at 0:28










                      • $begingroup$
                        Sorry, I should have specified that this is the answer for part b only. Part a is ok.
                        $endgroup$
                        – Andrei
                        Mar 6 at 0:30










                      • $begingroup$
                        @RobertShore yeah I am sorry. In my paper I showed that $ ysinfrac{1}{y}rightarrow 0 $
                        $endgroup$
                        – Dr.Mathematics
                        Mar 6 at 1:00
















                      1












                      $begingroup$

                      Use polar coordinates $x=rcostheta$ and $y=rsintheta$. The limit is the same as $rto 0$. The original trigonometric functions are always between $-1$ and $1$, $|costheta-sintheta|<2$, so your original expression is always less than $2r$ in absolute value.






                      share|cite|improve this answer









                      $endgroup$













                      • $begingroup$
                        For (a), you're not quite where you need to be. You need both that the limit of one term of your difference does not exist and also that the limit of the other term does exist. For example, $lim_{x to 0} (sin (1/x) - sin (1/x)) = 0$ even though neither term of the difference has a limit.
                        $endgroup$
                        – Robert Shore
                        Mar 6 at 0:28










                      • $begingroup$
                        Sorry, I should have specified that this is the answer for part b only. Part a is ok.
                        $endgroup$
                        – Andrei
                        Mar 6 at 0:30










                      • $begingroup$
                        @RobertShore yeah I am sorry. In my paper I showed that $ ysinfrac{1}{y}rightarrow 0 $
                        $endgroup$
                        – Dr.Mathematics
                        Mar 6 at 1:00














                      1












                      1








                      1





                      $begingroup$

                      Use polar coordinates $x=rcostheta$ and $y=rsintheta$. The limit is the same as $rto 0$. The original trigonometric functions are always between $-1$ and $1$, $|costheta-sintheta|<2$, so your original expression is always less than $2r$ in absolute value.






                      share|cite|improve this answer









                      $endgroup$



                      Use polar coordinates $x=rcostheta$ and $y=rsintheta$. The limit is the same as $rto 0$. The original trigonometric functions are always between $-1$ and $1$, $|costheta-sintheta|<2$, so your original expression is always less than $2r$ in absolute value.







                      share|cite|improve this answer












                      share|cite|improve this answer



                      share|cite|improve this answer










                      answered Mar 6 at 0:23









                      AndreiAndrei

                      13.8k21330




                      13.8k21330












                      • $begingroup$
                        For (a), you're not quite where you need to be. You need both that the limit of one term of your difference does not exist and also that the limit of the other term does exist. For example, $lim_{x to 0} (sin (1/x) - sin (1/x)) = 0$ even though neither term of the difference has a limit.
                        $endgroup$
                        – Robert Shore
                        Mar 6 at 0:28










                      • $begingroup$
                        Sorry, I should have specified that this is the answer for part b only. Part a is ok.
                        $endgroup$
                        – Andrei
                        Mar 6 at 0:30










                      • $begingroup$
                        @RobertShore yeah I am sorry. In my paper I showed that $ ysinfrac{1}{y}rightarrow 0 $
                        $endgroup$
                        – Dr.Mathematics
                        Mar 6 at 1:00


















                      • $begingroup$
                        For (a), you're not quite where you need to be. You need both that the limit of one term of your difference does not exist and also that the limit of the other term does exist. For example, $lim_{x to 0} (sin (1/x) - sin (1/x)) = 0$ even though neither term of the difference has a limit.
                        $endgroup$
                        – Robert Shore
                        Mar 6 at 0:28










                      • $begingroup$
                        Sorry, I should have specified that this is the answer for part b only. Part a is ok.
                        $endgroup$
                        – Andrei
                        Mar 6 at 0:30










                      • $begingroup$
                        @RobertShore yeah I am sorry. In my paper I showed that $ ysinfrac{1}{y}rightarrow 0 $
                        $endgroup$
                        – Dr.Mathematics
                        Mar 6 at 1:00
















                      $begingroup$
                      For (a), you're not quite where you need to be. You need both that the limit of one term of your difference does not exist and also that the limit of the other term does exist. For example, $lim_{x to 0} (sin (1/x) - sin (1/x)) = 0$ even though neither term of the difference has a limit.
                      $endgroup$
                      – Robert Shore
                      Mar 6 at 0:28




                      $begingroup$
                      For (a), you're not quite where you need to be. You need both that the limit of one term of your difference does not exist and also that the limit of the other term does exist. For example, $lim_{x to 0} (sin (1/x) - sin (1/x)) = 0$ even though neither term of the difference has a limit.
                      $endgroup$
                      – Robert Shore
                      Mar 6 at 0:28












                      $begingroup$
                      Sorry, I should have specified that this is the answer for part b only. Part a is ok.
                      $endgroup$
                      – Andrei
                      Mar 6 at 0:30




                      $begingroup$
                      Sorry, I should have specified that this is the answer for part b only. Part a is ok.
                      $endgroup$
                      – Andrei
                      Mar 6 at 0:30












                      $begingroup$
                      @RobertShore yeah I am sorry. In my paper I showed that $ ysinfrac{1}{y}rightarrow 0 $
                      $endgroup$
                      – Dr.Mathematics
                      Mar 6 at 1:00




                      $begingroup$
                      @RobertShore yeah I am sorry. In my paper I showed that $ ysinfrac{1}{y}rightarrow 0 $
                      $endgroup$
                      – Dr.Mathematics
                      Mar 6 at 1:00











                      0












                      $begingroup$

                      $sin{(1/x)sin(1/y)}$ is an interval and $x-y$ under the limit of $(x,y)to{(0,0)}$ is an interval too. if a function in a close interval is continuous, its valued field is an interval, but its converse proposition may be not true. for example, your function is a counterexample. so you should separate your problem into two cases with $f^{'}(0,0)$ and $lim_{(x,y)to{(0,0)}}f^{'}(x,y)$



                      the first limit exists but the second do not! for example rewrite your function to $F_{x}^{'}(x,y)=sin(1/x)sin(1/y)+(x-y)sin(1/y)cos(1/x)(-1/x^2)$



                      $F_{y}^{'}(x,y)=-sin(1/x)sin(1/y)+(x-y)sin(1/x)cos(1/y)(-1/y^2)$



                      divide both side with $sin(1/x)sin(1/y)$ to get



                      $1+(x-y)cos(1/x)(-frac{1}{x^{2}sin(1/x)})$



                      $-1+(x-y)cos(1/y)(-frac{1}{y^{2}sin(1/y))}$



                      since $f(x)=x^2{sin(1/x)}$ or equal to zero exists a limit at $f^{'}(0)$, but $lim_{xto{0}}f^{'}(x)=2xsin(1/x)-cos(1/x)$ do not exists, this result give a certain conclusion to your problem. thank you!






                      share|cite|improve this answer









                      $endgroup$


















                        0












                        $begingroup$

                        $sin{(1/x)sin(1/y)}$ is an interval and $x-y$ under the limit of $(x,y)to{(0,0)}$ is an interval too. if a function in a close interval is continuous, its valued field is an interval, but its converse proposition may be not true. for example, your function is a counterexample. so you should separate your problem into two cases with $f^{'}(0,0)$ and $lim_{(x,y)to{(0,0)}}f^{'}(x,y)$



                        the first limit exists but the second do not! for example rewrite your function to $F_{x}^{'}(x,y)=sin(1/x)sin(1/y)+(x-y)sin(1/y)cos(1/x)(-1/x^2)$



                        $F_{y}^{'}(x,y)=-sin(1/x)sin(1/y)+(x-y)sin(1/x)cos(1/y)(-1/y^2)$



                        divide both side with $sin(1/x)sin(1/y)$ to get



                        $1+(x-y)cos(1/x)(-frac{1}{x^{2}sin(1/x)})$



                        $-1+(x-y)cos(1/y)(-frac{1}{y^{2}sin(1/y))}$



                        since $f(x)=x^2{sin(1/x)}$ or equal to zero exists a limit at $f^{'}(0)$, but $lim_{xto{0}}f^{'}(x)=2xsin(1/x)-cos(1/x)$ do not exists, this result give a certain conclusion to your problem. thank you!






                        share|cite|improve this answer









                        $endgroup$
















                          0












                          0








                          0





                          $begingroup$

                          $sin{(1/x)sin(1/y)}$ is an interval and $x-y$ under the limit of $(x,y)to{(0,0)}$ is an interval too. if a function in a close interval is continuous, its valued field is an interval, but its converse proposition may be not true. for example, your function is a counterexample. so you should separate your problem into two cases with $f^{'}(0,0)$ and $lim_{(x,y)to{(0,0)}}f^{'}(x,y)$



                          the first limit exists but the second do not! for example rewrite your function to $F_{x}^{'}(x,y)=sin(1/x)sin(1/y)+(x-y)sin(1/y)cos(1/x)(-1/x^2)$



                          $F_{y}^{'}(x,y)=-sin(1/x)sin(1/y)+(x-y)sin(1/x)cos(1/y)(-1/y^2)$



                          divide both side with $sin(1/x)sin(1/y)$ to get



                          $1+(x-y)cos(1/x)(-frac{1}{x^{2}sin(1/x)})$



                          $-1+(x-y)cos(1/y)(-frac{1}{y^{2}sin(1/y))}$



                          since $f(x)=x^2{sin(1/x)}$ or equal to zero exists a limit at $f^{'}(0)$, but $lim_{xto{0}}f^{'}(x)=2xsin(1/x)-cos(1/x)$ do not exists, this result give a certain conclusion to your problem. thank you!






                          share|cite|improve this answer









                          $endgroup$



                          $sin{(1/x)sin(1/y)}$ is an interval and $x-y$ under the limit of $(x,y)to{(0,0)}$ is an interval too. if a function in a close interval is continuous, its valued field is an interval, but its converse proposition may be not true. for example, your function is a counterexample. so you should separate your problem into two cases with $f^{'}(0,0)$ and $lim_{(x,y)to{(0,0)}}f^{'}(x,y)$



                          the first limit exists but the second do not! for example rewrite your function to $F_{x}^{'}(x,y)=sin(1/x)sin(1/y)+(x-y)sin(1/y)cos(1/x)(-1/x^2)$



                          $F_{y}^{'}(x,y)=-sin(1/x)sin(1/y)+(x-y)sin(1/x)cos(1/y)(-1/y^2)$



                          divide both side with $sin(1/x)sin(1/y)$ to get



                          $1+(x-y)cos(1/x)(-frac{1}{x^{2}sin(1/x)})$



                          $-1+(x-y)cos(1/y)(-frac{1}{y^{2}sin(1/y))}$



                          since $f(x)=x^2{sin(1/x)}$ or equal to zero exists a limit at $f^{'}(0)$, but $lim_{xto{0}}f^{'}(x)=2xsin(1/x)-cos(1/x)$ do not exists, this result give a certain conclusion to your problem. thank you!







                          share|cite|improve this answer












                          share|cite|improve this answer



                          share|cite|improve this answer










                          answered Mar 24 at 10:34







                          user653679





































                              draft saved

                              draft discarded




















































                              Thanks for contributing an answer to Mathematics Stack Exchange!


                              • Please be sure to answer the question. Provide details and share your research!

                              But avoid



                              • Asking for help, clarification, or responding to other answers.

                              • Making statements based on opinion; back them up with references or personal experience.


                              Use MathJax to format equations. MathJax reference.


                              To learn more, see our tips on writing great answers.




                              draft saved


                              draft discarded














                              StackExchange.ready(
                              function () {
                              StackExchange.openid.initPostLogin('.new-post-login', 'https%3a%2f%2fmath.stackexchange.com%2fquestions%2f3136903%2fcontinuous-function-and-limit-fx-y-left-beginmatrix-x-y-sin-frac1x%23new-answer', 'question_page');
                              }
                              );

                              Post as a guest















                              Required, but never shown





















































                              Required, but never shown














                              Required, but never shown












                              Required, but never shown







                              Required, but never shown

































                              Required, but never shown














                              Required, but never shown












                              Required, but never shown







                              Required, but never shown







                              Popular posts from this blog

                              Magento 2 - Add success message with knockout Planned maintenance scheduled April 23, 2019 at 23:30 UTC (7:30pm US/Eastern) Announcing the arrival of Valued Associate #679: Cesar Manara Unicorn Meta Zoo #1: Why another podcast?Success / Error message on ajax request$.widget is not a function when loading a homepage after add custom jQuery on custom themeHow can bind jQuery to current document in Magento 2 When template load by ajaxRedirect page using plugin in Magento 2Magento 2 - Update quantity and totals of cart page without page reload?Magento 2: Quote data not loaded on knockout checkoutMagento 2 : I need to change add to cart success message after adding product into cart through pluginMagento 2.2.5 How to add additional products to cart from new checkout step?Magento 2 Add error/success message with knockoutCan't validate Post Code on checkout page

                              Fil:Tokke komm.svg

                              Where did Arya get these scars? Unicorn Meta Zoo #1: Why another podcast? Announcing the arrival of Valued Associate #679: Cesar Manara Favourite questions and answers from the 1st quarter of 2019Why did Arya refuse to end it?Has the pronunciation of Arya Stark's name changed?Has Arya forgiven people?Why did Arya Stark lose her vision?Why can Arya still use the faces?Has the Narrow Sea become narrower?Does Arya Stark know how to make poisons outside of the House of Black and White?Why did Nymeria leave Arya?Why did Arya not kill the Lannister soldiers she encountered in the Riverlands?What is the current canonical age of Sansa, Bran and Arya Stark?